LSAT and Law School Admissions Forum

Get expert LSAT preparation and law school admissions advice from PowerScore Test Preparation.

 Administrator
PowerScore Staff
  • PowerScore Staff
  • Posts: 8916
  • Joined: Feb 02, 2011
|
#40046
Complete Question Explanation
(The complete setup for this game can be found here: lsat/viewtopic.php?t=6800)

The correct answer choice is (E)

The answer to this Global question cannot be prephrased, requiring you to use the rules in order to eliminate four of the answer choices. Access to the four templates discussed earlier would facilitate this task tremendously.

Answer choice (A): This answer choice is incorrect, because S must be earlier than V. Thus, the first and second pieces on the CD cannot be R and V. None of the templates show this answer choice to be a possibility.

Answer choice (B): This answer choice is incorrect. If R and W were the first and second piece, respectively, we would be required to place W between R and T:
  • ..... ..... ..... ..... ..... R :longline: W :longline: T
This sequence would be in violation of the third rule, and therefore cannot be true. Also, none of the templates show this answer choice to be a possibility.

Answer choice (C): This answer choice is incorrect. If S and T were the first and second piece, respectively, we would be required to place T between S and R:
  • ..... ..... ..... ..... ..... S :longline: T :longline: R
This sequence would be in violation of the second rule, and therefore cannot be true. Also, none of the templates show this answer choice to be a possibility.

Answer choice (D): This answer choice is incorrect. If T and W were the first and second piece, respectively, we would be required to place W between T and R:
  • ..... ..... ..... ..... ..... T :longline: W :longline: R
As with answer choice (B), this sequence would be in violation of the third rule, and therefore cannot be true. Also, none of the templates show this answer choice to be a possibility.

Answer choice (E): This is the correct answer choice. If W and S were the first and second piece, respectively, the first rule is immediately satisfied (S :longline: V). The third rule is also satisfied, because W would be earlier than both R and T. To comply with the second rule, we simply need to ensure that T is later than both R and S. The resulting local diagram proves that answer choice (E) could be true, and is therefore correct:
PT73_Game_#1_#5_diagram 1.png
Of course, with templates answer choice (E) is even easier to validate, because in Template 2A W can be first, and S can be second:
PT73_Game_#1_#5_diagram 2.png
You do not have the required permissions to view the files attached to this post.

Get the most out of your LSAT Prep Plus subscription.

Analyze and track your performance with our Testing and Analytics Package.